HS 321 - Exam 1

Ace your homework & exams now with Quizwiz!

OPQ, Inc., has a net income of $650,000 before depreciation and has purchased one used piece of equipment during the year for $1,200,000. What Section 179 depreciation is allowable? a. $0 b. $650,000 c. $1,000,000 d. $1,200,000

b. $650,000 Explanation: Section 179 is limited to the lesser of $1,080,000 (for 2022) and OPQ's net income of $650,000.

Bob would like to invest in bonds and is considering either a taxable bond with an interest rate of 7% or a tax-exempt municipal bond of comparable risk and quality with an interest rate of 5%. His marginal tax rate is 24%. In order to help Bob compare these two bonds, compute the equivalent tax-free rate for the taxable bond. a. 5.25% b. 5.00% c. 5.32% d. 3.80%

c. 5.32% Explanation: The equivalent tax-free rate for the taxable bond is 5.32%.0.07 × (1 − 0.24) = 5.32%

All persons who participate in management will be protected from liability beyond their investment in each of the following entities EXCEPT: a. A C corporation b. An S corporation c. A limited liability company d. A limited partnership

d. A limited partnership Explanation: All shareholders and managers will be protected by limited liability in a C corporation and in an S corporation. All managers who are members of a limited liability company are also protected by limited liability. With a limited partnership, the general partner is the manager and has unlimited liability.

Which of the following activities is least likely to be classified as a hobby? a. The collection and restoration of classic cars that are not held for resale b. The collection and restoration of guitars that are not held for resale c. The raising of horses on a privately owned farm d. The opening of a brewery in an industrial park

d. The opening of a brewery in an industrial park. Explanation: The collection and restoration of classic cars and guitars are activities likely to be engaged in for personal pleasure since these items are not being held for resale in an attempt to make a profit. The raising of horses is commonly classified as a hobby. While the opening of the brewery might bring personal pleasure to the owner(s), breweries of this size are for-profit businesses, and the opening of the brewery in an industrial park further signals that this is an activity engaged in for the pursuit of profit, as opposed to brewing beer at home for personal consumption.

Ian's valuable gun collection was stolen from his home. He purchased the guns for $100,000, and they were worth $125,000 at the time they were stolen. He did not list them separately on his homeowners insurance policy, so he only received $1,000 from insurance. How much may he deduct as a casualty loss on his tax return if his adjusted gross income (AGI) is $100,000? a. $0 b. $83,900 c. $88,900 d. $99,000

a. $0 Explanation: Casualty losses that are not declared national disasters are no longer deductible (2017 Tax Cuts and Jobs Act).

Julio is starting a new business. His major concern is limiting his legal liability, but he is also concerned with the ability of the company to raise capital. At some point, he would like to be able to easily sell partial interests in the business to other investors or perhaps even take the company public, so he does not want to limit the number of potential owners. Julio will be actively working in the business and would like to receive the same tax-advantaged fringe benefits as other employees. Which of the following entities would best suit Julio's needs? a. A C corporation b. An S corporation c. An LLC d. A general partnership

a. A C corporation Explanation: Option (B) is incorrect because Julio does not want to limit the number of potential owners and an S corporation cannot have more than 100 owners. Option (C) is incorrect; if Julio is interested in taking the company public in the future, the LLC form would not be appropriate. Option (D) is incorrect because a partnership would not provide the limited liability in which Julio is interested. LLC members, partners, and greater-than-2% S corporation owners are not eligible to receive the same tax-advantaged fringe benefits as other employees. (They are taxed on the benefits they receive.)

Two years ago, Chloe purchased 100 shares of Pacific Journey, Inc., for $15,000. Unfortunately, the value of the shares has dropped to $10,000. Chloe's daughter, Emma, is heading off to college, and Chloe is tired of waiting for a return on the stock. Chloe gives the stock to Emma when it is worth $10,000 to help fund Emma's education. If Emma sells the shares 3 months after the transfer for $12,000, what is the amount and character of her gain or loss? a. $0 b. $2,000 short-term capital gain c. $3,000 long-term capital loss d. $3,000 short-term capital loss

a. $0 Explanation: Chloe gave loss property to her daughter. When a taxpayer transfers property with a loss to someone else and the recipient sells the property for an amount between the donor's adjusted basis and the fair market value of the stock on the date of the gift, no gain or loss is recognized. In this example, the no gain/no loss corridor is from $10,000 (the fair market value of the stock on the date of the gift) to $15,000 (the adjusted basis in the hands of the donor). If Emma sells the stock for any amount between $10,000 and $15,000, she is not required to recognize any gain and she is prohibited from recognizing any loss on the transaction. Since there is no gain or loss, there is no need to categorize the tax result as either long term or short term. This is an example of the double basis rule AKA the bifurcated basis rule.

How much in the form of Social Security and Medicare taxes in 2022 is a taxpayer who is an employee earning $150,000 responsible for paying? a. $11,289 b. $8,854 c. $11,475 d. $9,300

a. $11,289 Explanation: The Social Security wage base for 2022 is $147,000..6.2% × $147,000 = $9,114There is no wage base for Medicare FICA taxes, so all $150,000 is taxed at 1.45% to the employee. $150,000 × 1.45% = $2,175$9,114 + $2,175 = $11,289

Dorian, single and aged 42, has the following items of income and expense for the current tax year: - Wages: $60,000 - Interest: $1,000 - Inheritance: $50,000 - Alimony paid: $10,000 (2016 divorce decree) - Child support paid: $8,000 - Federal taxes paid: $5,000 - State income taxes paid: $2,000 - Medical expenses: $20,500 How much of the medical expenses will he deduct on Schedule A assuming a 7.5% AGI floor? a. $16,675 b. $20,500 c. $10,000 d. $3,825

a. $16,675 Explanation: He could deduct the excess of medical expenses over the 7.5% hurdle of adjusted gross income (AGI). $51,000 × .075 = $3,825$20,500 − $3,825 = $16,675

On January 1 of this year, Patti purchased a home worth $1.5 million with an interest-only mortgage of $1.2 million. She is currently only paying interest on the mortgage at the rate of 5% annually. What amount of qualified residence interest may she deduct as home mortgage interest on Schedule A of her individual income tax return this year? a. $37,500 b. $50,000 c. $55,000 d. $60,000

a. $37,500 Explanation: Since Patti purchased her home after December 15, 2017, the deduction for qualified residence interest is limited to the interest on $750,000 of principal. The calculation is simply the interest rate (5%) multiplied by the qualifying limit of $750,000; this equals $37,500.

Arnold and Phoebe have been married for 20 years and always file a joint return, but they never itemize their deductions. They have a gross income of $80,000 and deductions for adjusted gross income (AGI) in the amount of $5,000, but they do not have any children. Neither Arnold nor Phoebe are over the age of 65, and neither is blind. What is Arnold and Phoebe's taxable income for the current year? Assume their standard deduction is $25,1000 for the current tax year. a. $49,900 b. $54,900 c. $62,450 d. $75,000

a. $49,900 Explanation: Arnold and Phoebe's taxable income is equal to their gross income less deductions for adjusted gross income, less the greater of the standard deduction or itemized deductions. Therefore, their taxable income can be calculated as follows: For the Current Tax Year - Gross Income= $80,000 - Less Deductions for AGI = (- $5,000) - Adjusted Gross Income = $75,000 - Less Standard Deduction (given) = $25,100 - Less Personal Exemption = (−$0) - Taxable Income = $49,900

Liam paid $50,000 for a 30% interest in a general partnership in which he does not materially participate. Assuming his basis in the partnership is equal to his initial investment and the partnership loss for the year is $110,000, how much of the loss can Liam deduct? a. $0 b. $33,000 c. $50,000 d. $110,000

b. $33,000 Explanation: Even though Liam's loss is limited to $50,000 due to the basis limitation, his allocable share of the partnership loss is only $33,000 (30% x $110,000).

Two years ago, Diane purchased 100 shares of the Colbert Corporation for $15,000. On January 1 of this year, Diane reviews her investment portfolio and finds out she has had a very profitable year. To offset some of her gains, Diane sells her shares of the Colbert Corporation for $10,000. On January 25 of the same year, Diane reads a newspaper article indicating that the price of the Colbert Corporation is expected to increase substantially. Second-guessing the wisdom of selling her previous shares of Colbert stock, she purchases 100 shares of the Colbert Corporation for $8,000. What are the tax consequences for Diane this year? a. Diane will have a $5,000 realized, but not recognized, loss. b. Diane will have an $8,000 realized and recognized loss. c. Diane will have a $5,000 realized and recognized loss. d. Diane will have a $7,000 realized, but not recognized, loss.

a. Diane will have a $5,000 realized, but not recognized, loss. Explanation: Since Diane purchased and sold substantially identical securities within 30 days, a wash sale occurs. Her realized loss on the sale of the original shares is calculated as follows: - Amount Realized = $10,000 - Less: Adjusted Basis = (-$15,000) - Equals: Gain or Loss = (-$5,000) Due to the wash sale transaction, however, Diane will not be permitted to recognize the loss in the year it was incurred. Instead, the realized but unrecognized loss of $5,000 will be added to the basis of the replacement securities.

Frank and Gina are trying to calculate their gross income. Which of the following items should they exclude from their gross income? I. $75,000 in cash inherited by Gina from her mother II. $30,000 borrowed by Frank and Gina from First City Bank III. A $10,000 gain from the sale of Frank and Gina's boat IV. $600 of interest earned on a loan made by Frank to his cousin Michael a. I and II b. III and IV c. I, II, and III d. I, II, and IV

a. I and II Explanation: Inheritance is not income. Inherited cash or property is excluded from gross income, so Option I is correct. Borrowed money is also excluded from gross income, so Option II is also correct. Gain on the sale of assets (Option III) and interest income (Option IV) are both included in gross income.

Rylee gave her grandson, Kim, stock worth $500,000 this year, in addition to a previous gift equal to the annual gift tax exclusion amount. She purchased the stock for $250,000 several years earlier and believed the value would increase substantially in the near future. Since she had already used up her lifetime gift tax exemption in prior tax years, Rylee paid $200,000 in gift taxes on the transfer. If Kim sells the stock 6 months after the transfer for $750,000, which of the following statements is correct? a. Kim will realize a $400,000 long-term capital gain. b. Kim will realize a $250,000 long-term capital gain. c. Kim will realize a $500,000 long-term capital gain. d. Kim will realize a $250,000 short-term capital gain.

a. Kim will realize a $400,000 long-term capital gain Explanation: Kim received the stock by gift, so he qualifies for a carryover basis. Rylee's original basis in the property was $250,000. Since Rylee paid gift tax on the transfer, Kim is permitted to increase his basis by the portion of the gift tax paid that represents gain. The portion of the gift that represents gain is 50% ($250,000 appreciation in the property divided by $500,000 fair market value of the property as of the date of gift), and 50% of the gift taxes paid equals $100,000. Kim's basis is therefore $350,000. If he sells the stock 6 months after the transfer for $750,000, he will realize a $400,000 gain. Because Rylee transferred appreciated property to Kim, her holding period is added to Kim's holding period for the asset, transforming the gain into a long-term capital gain.

Morgan receives nonqualified stock options (NQSOs) with an exercise price of $50 when the stock is trading at $50. Morgan exercises these options 2 years after the date of the grant when the stock price is $65 per share. Which of the following statements is correct? a. Morgan will have W-2 income of $15 per share upon exercise. b. Morgan will have W-2 income of $65 per share upon exercise. c. Morgan will have $50 of alternative minimum taxable income (AMTI) upon exercise. d. Morgan's adjusted basis for regular income tax will be $15 at exercise.

a. Morgan will hae W-2 income of $15 per share upon exercise. Explanation: Morgan will have W-2 income of $15 per share upon exercise. $65 - $50 = $15

All of the following are tax credits that reduce the tax liability calculated on taxable income EXCEPT a. Student loan interest tax credit b. Child tax credit c. Earned income credit d. Retirement saver's credit

a. Student loan interest tax credit Explanation: There is no such thing as a student loan interest tax credit. All of the other items are credits against the calculated tax.

What is the usual federal income tax treatment of the employer's premium payments under a group long-term disability income insurance plan? a. The amount of the premium payments is excluded from the employee's income, regardless of whether or not the plan is discriminatory. b. The amount of the premium payments is fully included in the employee's income if the plan is discriminatory. c. The amount of the premium payments is included in the employee's income to the extent the plan will replace more than 50% of an employee's earnings. d. The amount of the premium payments is excluded from the employee's income only if the plan is nondiscriminatory.

a. The amount of the premium payments is excluded from the employee's income, regardless of whether or not the plan is discriminatory. Explanation: When group long-term disability insurance is paid for by the employer, the amount of the premium payments is excluded from the employee's income even if the plan is discriminatory.

Which of the following is NOT a requirement of the individual real estate investor exception to the passive-activity loss rules? a. The taxpayer must materially participate in the activity. b. The taxpayer must own at least 10% of the value of the real estate. c. The taxpayer must have an adjusted gross income (AGI) of less than $150,000. d. The taxpayer must actively participate in the activity.

a. The taxpayer must materially participate in the activity. Explanation: The taxpayer is not required to materially participate in the activity, but the taxpayer must actively participate in the activity. Material participation requires substantial, continuous involvement in the operation of the activity. Active participation means the taxpayer participates in making management decisions concerning the activity but is not substantially and continuously involved in its operation.

Grace is single, and her share of qualified business income (QBI) from a partnership is $95,000. Her total taxable income from all sources before taking the 20% deduction for QBI is $102,000. What is the amount of the Section 199A QBI deduction that Grace can take this year? a. $7,000 b. $16,600 c. $19,000 d. $20,400

c. $19,000 Explanation: The deduction is the lesser of 20% of QBI and 20% of taxable income. Twenty percent of her QBI is $19,000. Twenty percent of her taxable income is $20,400. Her deduction is $19,000.

Gina spent $5,000 in day care services for her three children so that she could work. Assuming her earned and adjusted gross income is $100,000, how much is her dependent care credit? (Please ignore changes from the American Rescue Plan Act of 2021.) a. $1,200 b. $1,000 c. $5,000 d. $3,000

b. $1,000 Explanation: The dependent care credit is not phased out, and it provides a credit of 20% on up to $3,000 per qualifying child with a maximum of $6,000 for two or more children. However, since Gina's actual expenses are limited to $5,000, her credit is calculated as $5,000 × 0.20 = $1,000.

Colin, aged 15, has $11,650 of earned income from a newspaper route and $1,800 of unearned income. How much is taxable to Colin at Colin's tax rate? a. $400 b. $1,400 c. $12,000 d. $13,450

b. $1,400 Explanation: Since Colin's unearned income is below $2,300 (2022), all of his income in excess of the standard deduction is taxed at his rate. His standard deduction is $11,650 + $400 (2022) = $12,050. His total income is $13,450 − $12,050 standard deduction = $1,400.

Kim raises and sells horses as a hobby. She also has income from riding lessons that are part of the hobby activity. - Income from Horse Sales = $10,000 - Income from Riding Lessons = $5,000 - Feeding Expenses = $8,000 - Boarding Expenses = $6,000 - Veterinary Expenses = $2,000 How much of the income is taxable to Kim? a. $5,000 b. $15,000 c. $0 d. -$1,000

b. $15,000 Explanation: This is a hobby activity. All income is taxable.

Marcus has a 30% interest in a limited partnership, for which he paid $25,000. The partnership loss for the year is $180,000. How much of the loss can Marcus deduct? a. $0 b. $25,000 c. $54,000 d. $60,000

b. $25,000 Explanation: Marcus's loss is limited to $25,000 due to the at-risk rules. He will also have a suspended loss of $29,000, that is, ($180,000 × 30%) − $25,000.

Clark invested $100,000 in an annuity contract many years ago. This year, Clark annuitized the contract. The insurance company agreed to pay Clark $520.83 per month for 20 years. Assuming that Clark receives eight payments this year, how much can Clark exclude from his gross income this year? a. $833.33 b. $3,333.31 c. $4,164.64 d. $6,249.96

b. $3,333.31 Explanation: Clark's expected return is $125,000, that is, 20 years × 12 months × $520.83. Therefore, his exclusion ratio is 80%, or $100,000 ÷ $125,000. Clark will receive $4,166.64 in annuity payments this year (8 payments × $520.83), of which $3,333.31 can be excluded. Therefore, Clark must include $833.33, or 20% × $520.83 × 8, in his income this year.

Bethann and Skyler, who are married filing jointly, have three children (aged 5, 9, and 17) who are dependents. The couple has a W-2 income of $115,000. They have $13,500 in taxes withheld. Bethann and Skyler have the following expenses for the year: - Interest on a home equity loan issued for $100,000 and used for secondary education = $2,500 - Interest on their first mortgage with an original loan balance of $450,000 = $12,000 - Property taxes on a personal residence = $8,000 - State income tax = $8,000 - State sales tax = $4,500 What is Bethann and Skyler's tax refund or tax due after all deductions and credits? (Assume a flat tax rate of 24% and that their standard deduction is $25,800.) (Please ignore changes from the American Rescue Plan Act of 2021.) a. $3,576 tax balance due b. $3,408 tax balance due c. $1,200 tax balance refund due d. $3,575 tax refund due

b. $3,408 tax balance due Explanation: $115,000 of gross income minus the standard deduction of $25,800 results in taxable income of $89,200. (The math: 0.24 × $89,200 = $21,408) Taxed at a rate of 24%, their tax liability is $21,408.

Eleanor loans $30,000 to her son James and does not charge any interest. James has investment income of $1,500 and investment expenses of $500. Assume that the applicable federal rate (AFR) is 1.1%. How much interest must be imputed on the loan? a. $1,500 b. $330 c. $2,000 d. $1,000

b. $330 Explanation: James has net investment income of $1,000. Therefore, the amount of imputed interest is the lesser of net investment income and the AFR-calculated interest minus the interest based on the stated rate of the loan. Since the stated rate of interest on the loan is 0%, the amount of imputed interest is the lesser of $1,000 and $330 (that is, $30,000 × 0.011). Therefore, $330 of interest must be imputed on the loan.

During her working years, Brianna ran an antiques store that was very successful. When she retired, she kept some of the display cases to display her own collections in her home. The display cases were originally purchased for the business at a cost of $15,000 and were fully depreciated by the time Brianna retired. Brianna died last month, and the display cases were valued in her estate at $8,000. If Brianna's daughter, Ingrid, inherits the display cases and sells them 2 months after Brianna's death for $8,500, what will be the income tax treatment on the sale? a. $500 ordinary income b. $500 long-term capital gain c. $500 short-term capital gain d. $8,000 ordinary income

b. $500 long-term capital gain Explanation: Even though the display cases were Section 1231 assets in Brianna's hands and were subject to depreciation recapture, once they ran through Brianna's estate, they qualified for a Section 1014 step to fair market value in basis, which eliminates the recapture potential. If Ingrid sells the cases for $8,500 2 months after Brianna's death, her gain will be $500 due to the step up in basis. Since all assets passing through an estate and receiving a step up in basis qualify for long-term capital-gains treatment, the gain will be characterized as a long-term gain.

In 2021, Lamark, a single-filing taxpayer, has an AMTI of $558,600. What is Lamark's AMT exemption this year? a. $35,000 b. $64,850 c. $73,600 d. $8,750

b. $64,850 Explanation: Because Lamark's AMTI is above the AMT phaseout threshold amount, his AMT exemption must be reduced. Lamark's exemption is reduced by 25% of the amount his AMTI exceeds $523,600 (the threshold). Therefore, Lamark's exemption must be reduced by $8,750, that is, ($558,600 − $523,600) × 0.25. As a result, Lamark is entitled to an exemption of $64,850, that is, $73,600 − $8,750.

Katie is a 15% owner in Enchanted Creations, LLC, a very successful web-developing business. She is also a 15% owner in Stronghold, LLC, an internet-based virus protection service. She materially participates in Enchanted Creations, but does not materially participate in Stronghold. She has an at-risk amount of $600,000 and her share of income generated by the company is $250,000 for Enchanted Creations. For Stronghold, she has an at-risk amount of $75,000 and her share of loss generated by the company is $200,000. What amount of loss is suspended because of the passive-activity income rules? a. $0 b. $75,000 c. $125,000 d. $200,000

b. $75,000 Explanation: Katie will not be allowed to take any loss for the current year. Of the $200,000 loss from Stronghold, $125,000 will be suspended because of the at-risk rules (she only has $75,000 at risk) and $75,000 will be suspended because of the passive-activity loss rules. (She does not have any passive income to offset the loss since she materially participates in Enchanted Creations.)

XYZ, Inc., has a net income of $2,650,000 before depreciation and has purchased two pieces of equipment during the year. The first piece of equipment cost $1,900,000, and the second piece of equipment cost $880,000. How much may be expensed under Section 179 for the current year for both pieces of equipment? a. $80,000 b. $970,000 c. $1,050,000 d. $2,700,000

b. $970,000 Explanation: The Section 179 expensing maximum is $1,080,000 (for 2022). This amount is reduced dollar-for-dollar by the amount of depreciable property placed in service during the year exceeding $2,700,000 (for 2022). The total placed in service was $2,780,000, which exceeds $2,700,000 by $80,000. The reduced limit is therefore $970,000 (or $1,050,000 − $80,000).

Contributions to charity are limited to a certain percentage of income. How long is the carry-over period for individuals to use any excess current charitable deductions? a. 1 year b. 5 years c. 7 years d. 15 years

b. 5 years Explanation: There is a 5-year carry-over provision for charitable deductions.

Joshua has three capital transactions during the current year: - A short-term capital loss (STCL) of $5,000 - A short-term capital gain (STCG) of $3,000 - A long-term capital loss (LTCL) of $2,000 What is the net effect on Joshua's income taxes if he is in the 35% income tax bracket? a. A $1,400 tax reduction b. A $1,050 tax reduction c. An $850 tax reduction d. A $450 tax reduction

b. A $1,050 tax reduction Explanation: The STCG and the STCL can be netted: Short-Term: Gains = $3,000 Losses = +$5,000 NSTCL = (-$2,000) Netting the $2,000 LTCL and the $2,000 STCL gives Joshua a total loss of $4,000. Joshua can only use $3,000 of losses to offset his ordinary income at 35% ($3,000 × 0.35 = $1,050). The remaining $1,000 is a long-term capital-loss carryover.

Yolanda became an AMT taxpayer last year. She had to add several items to her regular taxable income to arrive at her AMTI. Which of the following items will result in an AMT credit that can be used to offset future regular tax liability? a. $10,000 in state income taxes b. A $90,000 difference between the fair market value of stock and the strike price in the ISO used to purchase the stock c. $6,000 in interest on private activity municipal bonds d. $6,000 in home equity loan interest

b. A $90,000 difference between the fair market value of stock and the strike price in the ISO used to purchase the stock Explanation: The inclusion of the difference between the fair market value and exercise price of the stock options will result in a credit Yolanda can use against future regular income tax liability. Options (A) and (C) are adjustments, which result in permanent differences in tax liability as a result of the imposition of the AMT. Home equity loan interest is not deductible for regular tax or AMT.

The Tudor Company grants Edward one incentive stock option (ISO) on January 10, 2020. The exercise price is $10. The market price on the exercise date (June 12, 2021) is $33. What is the alternative minimum tax (AMT) consequence when Edward exercises the ISO? a. $0 AMT gain b. $10 AMT gain c. $23 AMT gain d. $33 AMT gain

c. $23 AMT gain Explanation: The AMT gain is the difference between the market price and the exercise price at the date of exercise.$33 − $10 = $23

Randy is starting a new business. He is concerned about legal liability. He would like to have flow-through taxation. At some point, he would like to be able to easily sell interests in the business, but he does not expect to have more than 20 investors. Randy does not want to pay self-employment taxes on all income. Which of the following entities would best suit Randy's needs? a. A sole proprietorship b. An S corporation c. A C corporation d. A partnership

b. An S corporation Explanation: Option (A) is incorrect because a sole proprietorship does not provide limited liability and Randy would need to change entities to take on future investors. Option (C) is incorrect because a C corporation would not provide flow-through taxation. Option (D) is incorrect because a partnership would not provide limited liability and Randy would need to pay self-employment taxes on the business net income. Only option (B), an S corporation, meets all of Randy's requirements.

John is a CPA, and Jack operates a laundromat. Instead of paying for each other's services in cash, they decide to exchange accounting services for laundromat services. The value of John's accounting service is $750, and the value of Jack's laundromat service is $600. Which of the following statements is correct? a. John must include $750 in his gross income. b. John must include $600 in his gross income. c. Jack must include $600 in his gross income. d. Neither must include any amount in gross income since barter transactions are tax-free.

b. John must include $600 in his gross income. Explanation: John must include $600 (the value of consideration received) in his gross income, and Jack must include $750 (the value of consideration received) in his gross income.

Which of the following would be added to a taxpayer's regular taxable income to arrive at their AMTI? a. Receipt of interest on public-purpose municipal bonds b. Receipt of interest on private activity municipal bonds c. Exercise of nonqualified stock options d. Sale of the shares purchased through the exercise of ISOs

b. Receipt of interest on private activity municipal bonds Explanation: While private activity municipal bonds generate interest that is exempt for regular income tax purposes, once a taxpayer is subject to the AMT, the interest generated by these bonds is taxable and must be added to regular taxable income to arrive at AMTI. The interest earned on public-purpose municipal bonds is always exempt from tax under the regular or AMT system. The exercise of a nonqualified stock option will generate ordinary income that is taxed for regular income tax purposes but would not need to be added back to calculate AMTI. The sale of shares purchased through the exercise of ISOs generates a negative adjustment to AMTI.

All the following statements concerning the AMT as it applies to individual taxpayers are correct EXCEPT: a. Some itemized deductions taken for regular tax purposes must be added back to income when calculating the AMT. b. Taxpayers are permitted to take the standard deduction for both regular and AMT tax purposes. c. All adjustments made to itemized deductions when calculating AMT result in a permanent increase in tax. d. Charitable deductions may be claimed in the same manner for regular and AMT tax purposes.

b. Taxpayers are permitted to take the standard deduction for both regular and AMT tax purposes. Explanation: Taxpayers who do not itemize deductions take the standard deduction for regular tax purposes, but this is added back to AMTI for AMT purposes. All of the other statements are correct.

The AMT was originally designed to a. create a more user-friendly tax system. b. curb abuses by high-income taxpayers. c. provide additional credits to certain low-income taxpayers. d. give taxpayers a choice of which tax to pay.

b. curb abuses by high-income taxpayers Explanation: The AMT was enacted in 1986 to curb perceived abuses by high-income taxpayers who were trying to minimize their current income tax liability.

During the current year, a taxpayer collected $600 interest on U.S. Treasury bills, $900 interest on a local municipal bond, and $200 from a state income tax refund. (She itemized her deductions last year.) She also received $400 in dividends from a U.S. common stock. Her gross income from the above is a. $400. b. $1,000. c. $1,200. d. $2,100.

c. $1,200 Explanation: Interest on the municipal bond is excluded under Section 103. Her gross income is $1,200, or $600 + $200 + $400. If the taxpayer had taken the standard deduction last year, she would not have included the state income tax refund since no benefit had been received.

John is a 30% partner in a partnership that produced business income (before adjustments) of $250,000 this year, and he is a 100% owner in an S corporation that produced net business income of $40,000 this year. The deduction for self-employment taxes allocated to his net business income is $5,299. His total taxable income is $110,000. What are John's qualified business income (QBI) and QBI deduction, respectively? a. $115,000 and $23,000 b. $115,000 and $22,000 c. $109,701 and $21,940 d. $110,000 and $22,000

c. $109,701 and $21,940 Explanation: John's QBI from the partnership is $69,701, that is, 30% × $250,000 − $5,299, and his QBI from the S corporation is $40,000. His total QBI is $109,701, that is, $69,701 + $40,000. His QBI deduction is $21,940, that is, 20% × $109,701.

Kelly, aged 14 and a dependent, has $2,600 in interest and dividends and $12,750 in earned income from a part-time job. What is Kelly's standard deduction in 2022? a. $2,300 b. $12,550 c. $12,950 d. $13,350

c. $12,950 Explanation: The standard deduction for a single person is limited to $12,950.

Cameron, an unmarried taxpayer using the single filing status, received $16,000 of Social Security retirement benefits this year. She also received $5,000 of interest income and $45,000 of income from her retirement plan during the year. How much of Cameron's Social Security benefits must be included in her gross income this year? a. $0 b. $8,000 c. $13,600 d. $16,000

c. $13,600 Explanation: Since her modified adjusted gross income (MAGI) of $50,000 plus one-half of her Social Security benefits (0.5 × $16,000 = $8,000) exceeds her adjusted base amount of $34,000, she must calculate her includible Social Security benefits using the lesser of the amount calculated using formula 3 or 4. Formula 3: 0.85 × $16,000 = $13,600 Formula 4: 0.85 × [$50,000 + (0.50 × $16,000) − $34,000] = $20,400 plus the least of $4,500, the amount calculated using formula 1, and the amount calculated using formula 2: Formula 1: 0.50 × $16,000 = $8,000 Formula 2: 0.50 × [$50,000 + (0.50 × $16,000) − $25,000] = $16,500 $4,500 is the least of these three numbers.The formula 4 total is $24,900, or $20,400 + $4,500. The lesser of the amounts for formula 3 and formula 4 is $13,600. Therefore, $13,600 of the Social Security benefits must be included in Cameron's gross income.

Gabriel and Anna got married and bought a house 15 months ago. Anna's job recently transferred her to an office in a different state, so Gabriel and Anna sold their house. What is the maximum amount of gain from the sale of the personal residence Gabriel and Anna can exclude from their gross income? a. $0 b. $250,000 c. $312,500 d. $500,000

c. $312,500 Explanation: Although they did not live in their house for a full 2 years, Gabriel and Anna are eligible for a prorated exclusion because of Anna's change in employment. They are eligible for a maximum exclusion of $312,500 (or ¹⁵⁄₂₄ × $500,000).

Petra, a married-filing-jointly taxpayer, paid $10,000 of qualified tuition and related expenses for each of her twin sons, Jamie and Graham, during the current year. They started their freshman year of college. Petra was very excited that both sons excelled in the college environment, especially since Jamie had a drug addiction during his senior year of high school. Petra had a friend on the college admissions board who thankfully overlooked Jamie's felony drug conviction. Petra also paid $2,000 of qualified tuition and related expenses for her son Tom's sophomore year of college and $3,000 for her own master's degree program. Petra claims all three of her sons as dependents. Her MAGI for the year is $50,000. What is the available American opportunity tax credit for the current year? a. $2,000 b. $2,500 c. $4,500 d. $7,500

c. $4,500 Explanation: The credit is not available for Jamie since he has a felony drug conviction. The maximum American opportunity tax credit of $2,500 is available to Graham. The credit available for Tom is $2,000, or $2,000 x 100%. Tom would have needed $4,000 of expenses to get the maximum credit. The total of credits for Graham and Tom is $4,500 for the current year.

Gavin, an unmarried individual who is an executive at IT Consulting, Inc., was granted 1,500 ISOs on IT's stock 2 years ago, when the price per share was $25. The last few years have resulted in tremendous growth for IT Consulting, and the stock is now trading at $55 per share. Gavin exercised the ISOs but did not sell the stock; he plans on holding the shares for at least a year so he can pay the lower capital-gains tax rate on the growth. How much will Gavin need to add to his taxable income when computing alternative minimum taxable income (AMTI) as a result of this transaction? a. $0 b. $37,500 c. $45,000 d. $82,500

c. $45,000 Explanation: Even though exercise of the options results in no taxable event for regular tax purposes this year, Gavin will need to add $45,000, that is, ($55 − $25) × 1,500, to his taxable income when computing AMTI.

Andrei, aged 45, is an active participant in his employer's defined-benefit retirement plan, but he would also like to make a deductible contribution to a traditional IRA this year. He is married, files a joint return with his wife, and has an adjusted gross income (AGI) of $112,000 in 2022. What is the maximum deductible contribution that Andrei can make to a traditional IRA? a. $900 b. $3,000 c. $5,100 d. $6,000

c. $5,100 Explanation: The phase-out range for taxpayers who are active participants and use the married-filing-jointly filing status is $109,000 to $129,000 for 2021. Since Andrei's AGI is within this range, he may not make a full $6,000 deductible contribution to a traditional IRA, but he may make a reduced deductible contribution, as calculated by the following formula: reduction = contribution limit × [(AGI − lower limit) ÷ $20,000)] Therefore, Andrei's deductible contribution is reduced by $900, or $6,000 × [($112,000 − $109,000) ÷ $20,000]. The maximum deductible contribution that Andrei can make to a traditional IRA is $5,100, or $6,000 − $900.

Karen and Jim, both aged 35, are married and filed a joint federal income tax return for 2022. Karen earned a salary of $138,000 and was covered by her employer's pension plan. Jim was not employed, and the couple had no other income. On February 15, Karen contributed $6,000 to an IRA for herself and $6,000 to an IRA for Jim. The allowable IRA deduction on Karen and Jim's 2022 joint tax return is a. $0. b. $3,000. c. $6,000. d. $12,000.

c. $6,000 Explanation: Karen's IRA is not deductible due to the regulations concerning the phaseout of deductibility of a taxpayer who is an active participant. The phaseout for married taxpayers filing jointly begins at $109,000 and ends at $129,000 for 2022. Jim's IRA is fully deductible. As the spouse of an active participant in a qualified retirement plan, his phaseout is $198,000 to $208,000 (2022). Therefore, the correct answer is $6,000, or Jim's contribution.

Wanda, an unmarried mother, has four children, aged 5, 9, 13, and 18. What is the maximum amount of her child tax credit for the current tax year assuming that she is under the adjusted gross income (AGI) threshold? (Please ignore changes from the American Rescue Plan Act of 2021.) a. $2,000 b. $4,000 c. $6,000 d. $8,000

c. $6,000 Explanation: The 18-year-old does not qualify since the cutoff for children is under, but not including, age 17. Therefore, Wanda will get a $2,000 tax credit for each of her three children, which will come to a total of $6,000.

A well-known sculptor donated one of her original bronze creations to a local charity, which auctioned the piece for $3,000. The artist's costs were as follows: - Bronze: $425 - Other materials: $150 - Pro rata overhead: $125 - Furnace/casting fees: $200 - Artistic contribution: $2,100 Assuming this is the only charitable contribution and based on an annual income of $150,000, what is the maximum amount of charitable income tax deduction available to the artist? a. $3,000 b. $2,100 c. $775 d. $900

c. $775 Explanation: Only materials and expenses are deductible, not artistic contribution or time. No deduction is allowed for use of property; therefore, the pro rata overhead would likely not be allowed.

Assume a taxpayer incurred business losses in Year 1 and generated an NOL of $20,000 that would be carried forward to offset income in future years. Assuming the taxpayer had $10,000 of taxable income and incurred no additional business income or loss in Year 2, what amount of NOLs can be recognized in Year 2? a. $0 b. $20,000 c. $8,000 d. $10,000

c. $8,000 Explanation: NOLs can only be carried forward and are limited to 80% of taxable income. In this case, 80% of taxable income ($10,000) is equal to $8,000. Therefore, only $8,000 of the NOL carryforward ($20,000) can be recognized in Year 2.

Assume a taxpayer incurred business losses in Year 1 and generated a Net Operating Loss (NOL) of $14,000 that would be carried forward to offset income in future years. Assuming the taxpayer had $14,000 of taxable income and incurred no additional business income or losses in Year 2, what amount of NOLs would be carried forward to Year 3? a. $0 b. $11,200 c. $14,000 d. $2,800

d. $2,800 Explanation: NOLs can only be carried forward and are limited to 80% of taxable income: 80% x $14,000 = $11,200. $11,200 of the $14,000 NOL carryforward may be recognized in Year 2 with the remainder ($2,800) being carried forward to Year 3.

All the following are requirements for the deferral of gain in a nonsimultaneous exchange under Section 1031 EXCEPT: a. The replacement property must be like-kind property with respect to the original property. b. The proceeds from the sale of the original property must be held by an escrow agent. c. A replacement property must be identified within 90 days of the sale of the original property. d. The closing on the replacement property must take place by the earlier of 180 days from the sale of the original property and the due date (including extensions) of the tax return for the year in which the original property was sold.

c. A replacement property must be identified within 90 days of the sale of the original property. Explanation: A replacement property must be identified within 45 days, not 90 days, of the sale of the original property.

Last year, Alessandra bought a home in Detroit. She is now considering selling her home and buying a new home, but she is not sure whether she can qualify for a prorated exclusion of the gain on the sale of her Detroit home under Section 121 of the Internal Revenue Code. Alessandra would qualify for a prorated exclusion under all the following circumstances EXCEPT: a. Alessandra has decided to sell her house because she has accepted a new job in New York City. Her last job was in Detroit. b. Alessandra has decided to sell her house because her personal physician recommended she move to the desert in Arizona because the air pollution in Detroit is causing her asthma to get worse. c. Alessandra has decided to sell her house because her dog, Tinkerbell, has arthritis and cannot walk down the stairs in the home. d. Alessandra has decided to sell her house because she is bothered by excessive noise from a nearby airport.

c. Alessandra has decided to sell her house because her dog, Tinkerbell, has arthritis and cannot walk down the stairs in the home. Explanation: The health of a pet is not considered a change in health that justifies a partial exclusion under Section 121. All the other options are reasons that would justify a partial exclusion under Section 121.

Ralph is not married and does not have any children. However, Ralph is a very good son and provides more than half of the cost of maintaining a very nice apartment for his mother and more than half of her support since her only income is a small amount from Social Security. Which of the following filing statuses should Ralph use, and why? a. Single because Ralph is not married b. Single because Ralph does not have any qualifying children c. Head of household because Ralph's mother is his dependent d. Head of household because Ralph's mother is a qualifying child

c. Head of household because Ralph's mother is his dependent Explanation: Ralph provides more than half of his mother's support and her income is below the exemption reference limit (which does not count Social Security income), so he can claim his mother as a dependent and is eligible to use the head-of-household filing status. Option (D) is incorrect because Ralph's mother is a qualifying relative, not a qualifying child. Options (A) and (B) are incorrect; since Ralph is eligible to use the head-of-household filing status, he should use that filing status rather than the less advantageous single filing status.

Kurt, Adrian, and Leonard each have an ownership interest in Three Guys' Hot Dogs, Inc. Based on the following information, which of them is considered to have materially participated in the conduct of the Three Guys' Hot Dogs business this year? I. Kurt dedicated more than 500 hours this year to Three Guys' Hot Dogs. II. Adrian devoted 150 hours to Three Guys' Hot Dogs this year. III. Leonard devoted 115 hours to Three Guys' Hot Dogs this year, but he also devoted more than 100 hours to several other activities for a total of 520 hours in all the activities combined. a. I only b. II and III c. I and III d. I, II, and III

c. I and III Explanation: Adrian has not materially participated. Although Adrian devoted more than 100 hours to the activity, he did not devote more hours than anyone else because Kurt worked at Three Guys' Hot Dogs for more than 500 hours. Leonard is also a material participant because he devoted more than 100 hours to the activity and also devoted more than 100 hours to several other activities for a total of more than 500 hours in all of the activities combined.

Bryan and Diane are trying to calculate their gross income. Which of the following items should they exclude from their gross income? I. A $25,000 gift from Diane's mother for the down payment of their new house II. $30,000 borrowed by Bryan and Diane from First City Bank III. A $10,000 increase in the value of Delta Airlines stock, which they own in their brokerage account IV. $55,000-worth of home repair work that was exchanged for tax work by Bryan a. I and II b. III and IV c. I, II, and III d. I, II, and IV

c. I, II, and III Explanation: Gifts are not income. A gift of cash or property is excluded from gross income. Borrowed money is also excluded from gross income. Gain on assets is not taxable until the assets are sold. However, barter transactions are taxable.

Tyrone owns a cottage that he sometimes rents out to other people. Which of the following statements regarding Tyrone's rental activity is correct? a. If Tyrone rents out the cottage for less than half of the year, it will be considered a nontaxable activity. b. If Tyrone rents out the home for exactly half the year and uses the home personally for exactly half the year, the activity will be considered primarily a rental activity. c. If Tyrone rents out the home for 180 days per year and uses the home personally for 20 days of the year, the activity will be considered a mixed-use activity. d. If Tyrone rents out the home for more than half of the year, it will be considered primarily a rental activity regardless of his personal use.

c. If Tyrone rents out the home for 180 days per year and uses the home personally for 20 days of the year, the activity will be considered a mixed-use activity. Explanation: If Tyrone rents out the home for 180 days per year and uses the home personally for 20 days of the year, the activity will be considered a mixed-use activity. Option (A) is not correct because the activity will only be considered a nontaxable activity if Tyrone rents out the cottage for less than 15 days per year. Option (B) is not correct because this option meets the requirements of a mixed-use activity. That is, the property is rented out for 15 days or more, but the owner personally uses it for the greater of 14 days or 10% of the rental days. Option (D) is not correct because even if Tyrone rents out the home for more than half of the year, the activity may be a mixed-use activity if he personally uses the property for the greater of 14 days and 10% of the rental days.

Precision Turbo Mechanics (PTM), Inc., purchased a new machine for cleaning and retooling the turbo blades on trucks. The machine cost $30,000, and there was a 10% sales tax and a $1,000 delivery and setup fee. What is PTM's basis in the new machine? a. $30,000 b. $31,000 c. $33,000 d. $34,000

d. $34,000 Explanation: PTM's basis equals the machine cost of $30,000 plus the sales tax of $3,000 and the $1,000 delivery and setup fee.

Payments for employment-related care that are made to relatives of the taxpayer may qualify for the credit for child- and dependent-care expenses. Which of the following payments does not qualify? a. Payments for employment-related care made to the taxpayer's aunt b. Payments for employment-related care made to the taxpayer's 21-year-old daughter (who is not a dependent of the taxpayer) c. Payments for employment-related care made to a dependent of the taxpayer d. Payments for employment-related care made to the taxpayer's 17-year-old niece

c. Payments for employment-related care made to a dependent of the taxpayer Explanation: Payments for employment-related care made to a dependent of the taxpayer do not qualify for the credit for child- and dependent-care expenses. All the other options are qualifying payments.

Which of the following is not a deduction for adjusted gross income (AGI)? a. Maintenance expenses for a rental property actively managed by the taxpayer b. Moving expenses of a taxpayer who is an active-duty member of the Armed Forces c. Real estate taxes d. One-half of self-employment tax paid

c. Real estate taxes Explanation: Real estate taxes are deductible from AGI and are limited to $10,000 after 2017; they are a below-the-line deduction. Moving expenses are a deduction for AGI for members of the Armed Forces. Self-employed taxpayers can receive a deduction for one-half of self-employment taxes paid, and maintenance expenses for an actively managed rental property are deducted for AGI.

All the following are disadvantages of the sole proprietorship as a business form EXCEPT: a. The ability of the sole proprietor to raise capital is usually limited to their own resources. b. The transferability of the business is severely limited. c. There are many requirements to starting a sole proprietorship that make it difficult to get this type of business up and running. d. A sole proprietor has unlimited liability for business debts.

c. There are many requirements to starting a sole proprietorship that make it difficult to get this type of business up and running. Explanation: There are practically no requirements that make the initiation of a sole proprietorship difficult. However, a sole proprietor commonly has a problem raising large amounts of capital, and the transferability of the business is usually limited because the input of the proprietor themself is the factor that makes the business viable and valuable. On top of all of these issues is the unlimited liability the sole proprietor has for their business debts.

Tony's son, John, completed his junior year of college in 2021. Tony paid $3,000 in qualified expenses for John in 2021. Tony is a married-filing-jointly taxpayer and has an adjusted gross income (AGI) of $150,000 for the current year. What education credit will provide Tony the highest credit, and how much is that credit? a. Tony can claim a lifetime learning credit in the amount of $600. b. Tony can claim a lifetime learning credit in the amount of $3,000. c. Tony can claim an American opportunity tax credit in the amount of $2,250. d. Tony can claim an American opportunity tax credit in the amount of $2,500.

c. Tony can claim an American opportunity tax credit in the amount of $2,250. Explanation: Tony can claim an American opportunity tax credit in the amount of $2,250.($2,000 × 100%) + ($1,000 × 25%) = $2,250

All the following statements concerning health savings accounts (HSAs) are correct EXCEPT: a. Excess contributions to HSAs are subject to a 6% penalty tax. b. When a self-employed individual makes a contribution to an HSA, the contribution is disregarded when calculating self-employment taxes for Social Security. c. Unless contributions to HSAs are distributed to cover health care costs by the end of the taxable year, they are forfeited. d. Distributions from HSAs that are not used to cover medical expenses are subject to a 20% penalty until the taxpayer reaches age 65.

c. Unless contributions to HSAs are distributed to cover health care costs by the end of the taxable year, they are forfeited. Explanation: Unlike for health care flexible spending accounts (which are accounts maintained by employers that allow employees to allocate part of their income on a pretax basis to the account), amounts contributed to HSAs that are not used by the end of the taxable year can be carried forward and used in future years. Excess contributions to HSAs are subject to a 6% penalty tax, and early distributions used for nonqualified expenses (before age 65) are subject to a 20% penalty tax. Furthermore, contributions to HSAs are not taken into consideration when determining the amount of income subject to Social Security taxes for self-employed individuals.

This year, Serenity's aggregate income from her businesses was $180,000. Serenity's aggregate business deductions from those businesses were $320,000. Serenity's spouse, Tristan, earns $80,000 of W-2 income. The couple also has $20,000 of investment income. What amount of business loss may be used to offset the income from Tristan's earnings and the investment income? a. $0 b. $20,000 c. $80,000 d. $100,000

d. $100,000 Explanation: The business deductions total $320,000 and the business income is $180,000 resulting in a total business loss of $140,000. This is less than the $500,000 (adjusted for inflation) maximum threshold for married taxpayers filing jointly. As a result, Tristan's income ($80,000) and the investment income ($20,000) can be fully offset by the business loss.

Zihan recently purchased a new machine for his business. The price of the machine was $12,000, but Zihan also paid $100 in sales tax, $300 in freight, and $1,000 in installation and testing costs. What is Zihan's basis in the new machine? a. $12,100 b. $12,400 c. $13,000 d. $13,400

d. $13,400 Explanation: All costs to get the asset into operation are included in the cost basis of the asset. Therefore, Zihan's cost basis includes the price of the machine and the costs for sales tax, freight, and installation and testing.

Megan purchased series EE savings bonds for $4,000 in 1992 when she was 26. This year, she redeemed the bonds for $8,000 and paid qualified higher education expenses in the amount of $5,000 for her daughter. How much interest can Megan exclude from her gross income this year? a. $0 b. $4,000 c. $1,500 d. $2,500

d. $2,500 Explanation: Because Megan did not use all the proceeds from the bond redemption to pay for qualified education expenses, she will be required to include part of the interest income from the bonds in her gross income. Megan may exclude $2,500 of interest income from her gross income; that is, ($5,000 ÷ $8,000) × $4,000. Therefore, Megan must include $1,500 in her gross income, or $4,000 − $2,500.

Bill and Jen Walters purchased their home in 2007 and took out a mortgage of $1,000,000. The current loan balance is $875,000, and the fair market value of the home is $1,400,000. The rate on their existing loan is 6%, and the current rate for home loans is 4.5%, so the Walters would like to refinance to take advantage of the lower interest rate. If the Walters refinance the full $875,000 balance on their mortgage on January 1 of this year, what amount of interest is deductible as an itemized deduction this year? a. $0 b. $33,750 c. $34,453 d. $39,375

d. $39,375 Explanation: Their original mortgage was taken out before December 15, 2017, allowing them to retain the right to deduct the interest on up to $1,000,000 of debt so long as they do not refinance more than the existing acquisition loan balance. $875,000 × 4.5% = $39,375

Helen is covered by a group term life insurance policy that provides coverage equal to three times her annual salary of $125,000. Helen's employer pays the entire cost of the policy, for which the uniform premium is $0.15 per $1,000 of coverage per month. How much of this premium is taxable to Helen? a. $1,500 b. $1,000 c. $48.75 d. $585

d. $585 Explanation: The cost of up to $50,000 of coverage is nontaxable. $375,000 − $50,000 = $325,000[($325,000 ÷ $1,000) × $0.15] × 12 months = $585 taxable coverage

Corinne is an unmarried taxpayer with three children at the ages of 2, 4, and 6. Corinne's modified adjusted gross income (MAGI) is $85,000. What is the amount of the child tax credit that Corinne can claim this year? a. $0 b. $1,000 c. $2,500 d. $6,000

d. $6,000 Explanation: Corinne has three children under the age of 17. The child tax credit is $2,000 per child. While the allowable tax credit amount is reduced by $50 for each $1,000 that a single-filing taxpayer's modified adjusted gross income exceeds $200,000 (TCJA 2017), Corinne's MAGI does not exceed the threshold. Therefore, Corinne can claim a total of $6,000 under the child tax credit, or $2,000 × 3.

Henry, single and aged 42, was divorced after 12/31/2018. He had the following items of income and expense for the current tax year: Wages: $60,000 Interest: $1,000 Inheritance: $50,000 Alimony paid: $10,000 Child support paid: $8,000 Federal taxes paid: $5,000 State income taxes paid: $2,000 Medical expenses: $7,500 What is Henry's adjusted gross income (AGI)? a. $49,000 b. $51,000 c. $59,000 d. $61,000

d. $61,000 Explanation: His income includes the wages and interest. The alimony paid is not deductible for AGI if the divorce decree is post 12/31/2018. Therefore, his AGI is $61,000.$60,000 + $1,000 = $61,000

Charlotte, aged 70, is single and an employee of Interstellar Corporation. Her only sources of income this year are $80,000 of W-2 wages, $6,000 in Social Security benefits, and $1,000 of interest on State of Georgia bonds. Based on this information, Charlotte's adjusted gross income (AGI) for the current year is a. $80,000. b. $81,000. c. $83,000. d. $85,100.

d. $85,100 Explanation: Charlotte's adjusted gross income can be calculated as follows: - W-2 Income = $80,000 - Interest on State of Georgia bonds (tax-exempt) = -0- - Social Security Benefits (85% × $6,000) = + 5,100 - $85,100 Her Social Security benefits are 85% taxable because of her modified AGI.

Jennifer, who is single, is a partner in a law firm with one other individual. Her share of partnership income is $75,000 and her total taxable income (excluding the qualified business income deduction) is $285,000, placing her in the 35% marginal tax bracket. What is Jennifer's adjusted marginal tax rate on the partnership income after factoring in the Section 199A qualified business income (QBI) deduction? a. 7% b. 24% c. 28% d. 35%

d. 35% Explanation: Jennifer is in the 35% bracket, and her income is above the phaseout range for a specified service trade or business (SSTB). As a result, she will not be eligible for the QBI deduction.

Trevor passed away 2 months after he purchased Sunnydale for $30,000. The fair market value of Sunnydale as of the date of Trevor's death was $32,000. He left Sunnydale to his son, Kirk. Since Kirk was the only beneficiary of the estate and there were no estate taxes due, the title to the property was transferred to Kirk within one month of Trevor's death. Two weeks after receiving title to the property, Kirk sold Sunnydale for $35,000. What is the amount and type of income Kirk will report on the sale? a. A $5,000 short-term capital gain b. A $5,000 long-term capital gain c. A $3,000 short-term capital gain d. A $3,000 long-term capital gain

d. A $3,000 long-term capital gain Explanation: When Trevor died, the basis of Sunnydale qualified for a step to fair market value under IRC Section 1014. Kirk's basis in the property is therefore $32,000. Since he sold the property for $35,000, Kirk's gain is $3,000. Even though Trevor purchased the property 3.5 months before it was sold, Kirk's gain will be a long-term capital gain. Any property received through the estate of a decedent automatically qualifies for long-term capital-gains treatment.

Which type of distribution qualifies for an exception to the 10% early withdrawal penalty if made from either a qualified retirement plan or IRA? a. A distribution used to pay for qualifying higher education expenses b. A distribution used to pay for qualifying K-12 tuition expenses c. A distribution taken after reaching age 55 and separating from an employer d. A distribution taken to pay for qualifying medical expenses that exceed a percentage of AGI

d. A distribution taken to pay for qualifying medical expenses that exceed a percenage of AGI. Explanation: Option (A) only applies to distributions from IRAs. Option (B) does not apply to distributions from either qualified plans or IRAs. Option (C) only applies to distributions from qualified plans.

Which of the following is a risk faced by the owner of a business entity? a. The death or disability of another owner b. The retirement of another owner c. Bankruptcy d. All of the above

d. All of the above Explanation: All the options are potential risks faced by the owner of a business entity.

Toni owns a downtown office building. She originally purchased the building for $1,500,000 and took straight-line depreciation deductions of $800,000. What will be the tax consequences if Toni sells the building for $1,600,000? (Ignore the Medicare tax on net investment income.) a. Toni will have ordinary income of $0. b. Toni will have $800,000 of gain taxed at 25%. c. Toni will have a Section 1231 gain of $100,000 taxed at preferential long-term capital-gains tax rates. d. All of the above will occur.

d. All of the above will occur. Explanation: - Sale Price = $1,600,000 - Less Adjusted Basis (that is, $1,500,000 - $800,000) = (-$700,000) - Equals Gain = $900,000 Breakdown of gain: - Toni will have a $800,000 unrecaptured Section 1250 gain (straight-line depreciation). This is net capital gain taxed at regular rates of up to 25%. - Toni will have a $100,000 net capital gain taxed at long-term capital-gains tax rates.

On January 1, 2018, Axel and Sasha divorced. Under the terms of the divorce decree, Sasha was given custody of their only child, Marcy, who is 16 years old. The court decree also requires Axel to pay Sasha $1,000 a month in alimony for 2 years (that is, until Marcy turns 18), followed by $800 a month for the 4 years after that. Which of the following statements concerning the payments that Axel makes to Sasha is correct? a. Axel will be able to reduce his adjusted gross income (AGI) by the $12,000 in alimony payments he makes to Sasha this year. b. Sasha will not be required to report any of the payments that Axel makes to her as income on her tax return. c. Axel will not be able to take a deduction for the alimony payments made due to the alimony recapture rule. d. Axel's tax deduction will be limited to $9,600 per year during the 6-year period.

d. Axel's tax deduction will be limited to $9,600 per year during the 6-year period. Explanation: For income tax purposes, alimony payments that are reduced when a minor child reaches the age of majority (18 in most states) are considered to be a form of child support. Since Axel's alimony payments will be reduced by $200 per month once Marcy reaches age 18, $200 of the payment will be considered child support, leaving the remaining $800 as alimony. As a result, Axel will only be able to deduct from his income, and Sasha will be required to include in her income, $9,600 per year.

All the following statements regarding depreciation recapture are correct EXCEPT: a. Applying depreciation recapture is not optional. b. Depreciation recapture applies to Section 1231 assets under Section 1245 and Section 1250. c. Depreciation recapture exists to offset the benefit the taxpayer previously received from claiming deductions that reduced his or her ordinary income. d. Depreciation recapture only applies to assets sold within 5 years of their original purchase.

d. Depreciation recapture only applies to assets sold within 5 years of their original purchase. Explanation: Statements (A) through (C) are all correct. Statement (D) is incorrect because depreciation recapture applies to all assets regardless of when the asset is sold and regardless of whether or not depreciation was previously claimed.

Doug and Katie are trying to calculate their gross income for the current year. Which of the following items should they include in their gross income? I. Dividends earned on a stock owned by Doug that were reinvested II. Money borrowed by Doug and Katie from Chase Bank III. Gain from the sale of Doug's classic guitar collection IV. Interest earned on a loan made by Katie to her sister a. I and II b. III and IV c. I, II, and III d. I, III, and IV

d. I, III, and IV Explanation: Dividends received, even if reinvested, are included in income, and therefore, Option I is correct. Borrowed money is excluded from gross income, and therefore, Option II is incorrect. Gain on the sale of assets (Option III) and interest income (Option IV) are both included in gross income.

Jen and Carlos purchased a vacation home 12 years ago for $250,000. Now, the couple is getting divorced. As part of the divorce settlement, Jen receives the vacation home, which is now worth $1,000,000. Which of the following statements is true? a. If Jen sells the vacation home 6 months after receiving it in the divorce settlement, any gain or loss she has will be short term. b. If Jen sells the vacation home for $800,000, she will have a $200,000 loss. c. In any future sale of the vacation home, Jen and Carlos will each have a basis of $125,000. d. If Jen sells the vacation home for $1,100,000, she will have a gain of $850,000.

d. If Jen sells the vacation home for $1,100,000, she will have a gain of $850,000 Explanation: Option (A) is incorrect because Jen has a carryover holding period, so if she sells the vacation home six months after receiving it in the divorce settlement, any gain or loss she has will be long term. Option (B) is incorrect because Jen has a carryover basis in the property; therefore, her basis is $250,000. Option (C) is incorrect because Carlos no longer has an interest in the property.

When an employee of the ABC Company elects salary reduction for a flexible spending account (FSA), which of the following statements is correct? a. Under the FSA, dependent care benefits can be provided for up to two children. b. The FSA is typically funded by employee and employer contributions. c. The salary reduction is subject to Social Security taxes but not to income taxes. d. If the employee does not use all the health care salary reduction, the balance is forfeited.

d. If the employee does not use all the health care salary reduction, the balance is forfeited. Explanation: If the employee does not use all the health care salary reduction, the balance is forfeited. (Note: While plans that do not allow an extension through March of the following year can now be amended to allow up to $500 from a health FSA to carry over, the general rule is still "use it or lose it.") Dependent care benefits are limited by dollar amount, not by number of children. The FSA is typically funded only by employee contributions. The salary reduction is not subject to Social Security or income taxes.

All the following statements regarding the deduction of costs associated with investigating the purchase of a new line of business are correct EXCEPT: a. If the new line of business is not purchased, no deduction is permissible. b. If the new line of business is purchased and it is in the same line of business as the current trade or business operation, the cost of investigating the new business is fully deductible. c. The ability to deduct the cost of investigating a new line of business is often overlooked by taxpayers. d. If the new line of business is purchased and it is in a different line of business than the current trade or business operation, the costs of investigation cannot be deducted.

d. If the new line of business is purchased and it is in a different line of business than the current trade or business operation, the costs of investigation cannot be deducted. Explanation: If the new line of business is purchased and it is in a different line of business than the current trade or business operation, the costs of investigation are subject to capitalization rules as a new line of business. In the case of a new (and unrelated) line of business, up to $5,000 of start-up costs (reduced by the amount that start-up expenditures exceed $50,000 but not to the point where the start-up costs fall below $0) can be deducted during the first year of operations.

Vinnie owns a life insurance contract. He recently reassessed his insurance needs and decided he would like to exchange his current life insurance contract for a different insurance product. Which of the following transactions might result in gain realization and recognition? a. Vinnie trades his life insurance contract for a different life insurance policy. b. Vinnie trades his life insurance contract for a modified endowment contract. c. Vinnie trades his life insurance contract for an annuity. d. None of the above transactions would result in the realization and recognition of gain.

d. None of the above transactions would result in the realization and recognition of gain. Explanation: Vinnie can trade his life insurance contract for another life insurance contract, a modified endowment contract, or an annuity and be eligible for the deferral of gain under Section 1035 of the Internal Revenue Code. Vinnie may not, however, exchange an annuity product for a life insurance product per the Section 1035 rules.

Which of the following is true regarding real estate activities? a. Real estate activities are always passive. b. An individual investor in rental real estate can always consider her real estate activities as active businesses. c. Closely held C corporations that participate in real estate activities will always be considered active businesses. d. Real estate professionals may be allowed to consider their real estate activities as active in some circumstances.

d. Real estate professionals may be allowed to consider their real estate activities as active in some circumstances. Explanation: Real estate activities are generally passive, but exceptions do apply. An individual investor in rental real estate will generally be subject to the passive-income rules; however, some exceptions apply, such as the real estate professional exception that applies to taxpayers who spend more than half of their time during the year materially participating in real estate activities and meet the 750-hour requirement. Closely held C corporations are also eligible if more than 50% of the gross receipts of the corporation are derived from real property trades or businesses in which the corporation materially participates.

LMN, Inc., has a net income of $650,000 before depreciation and has purchased one used piece of equipment during the year for $1,200,000. Which of the following depreciation strategies will allow LMN to claim the largest depreciation deduction in the current year? a. The use of Section 179 b. The use of the modified accelerated cost recovery system (MACRS) c. The use of Section 179 and MACRS d. The use of bonus depreciation

d. The use of bonus depreciation Explanation: Bonus depreciation allows a 100% write-off against income, whereas Section 179 is limited to the lesser of $1,080,000 (for 2022) and the net income of $650,000. While MACRS allows depreciation to be claimed on an accelerated basis, it will take 5 years to recover the full cost of the equipment according to the depreciation tables.


Related study sets

Chapter 10- Greek and Roman Theatre

View Set

Massage Principles & Practice Review

View Set

Funds of nursing module 17 geriatric quiz

View Set

Ch 31 Disorders of Ventilation and Gas Exchange

View Set